What Is the Correct Approach to Integrate 2*arctan(x) by Parts?

Click For Summary
The discussion focuses on the integration of the function 2*arctan(x) using integration by parts. The initial approach involved setting u = arctan(x) and v = x, leading to a complicated expression that ultimately returned to the original integral. A suggestion was made to avoid integration by parts and instead use the substitution u = 1 + x^2, which proved to be more effective. The confusion arose from the incorrect application of integration by parts, where the chosen u and dv led to a circular result. The correct method clarified that integration by parts was not suitable for this particular integral.
PCSL
Messages
146
Reaction score
0

Homework Statement


problem: \int2arctanx dx
2\intarctan dx

u=arctanx
du=1/(1+x2)
v=x
dv=dx

xarctanx-\intx/(1+x2)

integrate by parts a second time...

u=x
du=dx
v=arctanx
dv=1/1+x2

xarctanx-\intarctanx

My final answer I get it 2xarctanx-2xarctanx+2/x2+1 which is obviously wrong. Thanks.
 
Physics news on Phys.org
PCSL said:

Homework Statement


problem: \int2arctanx dx
2\intarctan dx

u=arctanx
du=1/(1+x2)
v=x
dv=dx

xarctanx-\intx/(1+x2)

Don't go integration by parts here, just do a substitution of u=1+x2
 
rock.freak667 said:
Don't go integration by parts here, just do a substitution of u=1+x2
Thanks, I got the answer. This may seem like a dumb question, but how come integration by parts didn't work for this part?
 
PCSL said:

Homework Statement


problem: \int2arctanx dx
2\intarctan dx

u=arctanx
du=1/(1+x2)
v=x
dv=dx

xarctanx-\intx/(1+x2)

integrate by parts a second time...

u=x
du=dx
v=arctanx
dv=1/1+x2

xarctanx-\intarctanx

My final answer I get it 2xarctanx-2xarctanx+2/x2+1 which is obviously wrong. Thanks.[/QUOTE]
Because 2/(x^ 1) is the <b>derivative</b> of arctan(x), not the integral.<br /> <br /> Your choice of &quot;u&quot; and &quot;dv&quot; are just the results you got from the first integration by parts so you are just reversing the first integration. What you would correctly get is <br /> x arctan(x)- x arctan(x)+ \int arctan(x)dx= \int arctan(x)dx<br /> exactly what you started with.
 
Question: A clock's minute hand has length 4 and its hour hand has length 3. What is the distance between the tips at the moment when it is increasing most rapidly?(Putnam Exam Question) Answer: Making assumption that both the hands moves at constant angular velocities, the answer is ## \sqrt{7} .## But don't you think this assumption is somewhat doubtful and wrong?

Similar threads

  • · Replies 22 ·
Replies
22
Views
3K
  • · Replies 5 ·
Replies
5
Views
2K
  • · Replies 15 ·
Replies
15
Views
2K
Replies
3
Views
1K
Replies
3
Views
2K
Replies
6
Views
2K
  • · Replies 1 ·
Replies
1
Views
2K
  • · Replies 1 ·
Replies
1
Views
1K
  • · Replies 3 ·
Replies
3
Views
2K
Replies
1
Views
2K